You are on page 1of 82

Texas Examinations

of Educator Standards™
(TExES™) Program

Preparation
Manual
Principal (068)

Copyright © 2015 by Texas Education Agency (TEA). All rights reserved. The Texas Education Agency logo
and TEA are registered trademarks of Texas Education Agency. Texas Examinations of Educator Standards,
TExES and the TExES logo are trademarks of Texas Education Agency.
Table of Contents
About The Test ........................................................................................... 3

The Domains .............................................................................................. 4

The Standards ............................................................................................ 5

Domains and Competencies.......................................................................... 6

Domain I — School Community Leadership ........................................... 6

Domain II — Instructional Leadership ................................................... 8

Domain III — Administrative Leadership ............................................... 11

Approaches to Answering Multiple-Choice Questions ........................................ 13

How to Approach Unfamiliar Question Formats ...................................... 13

Question Formats............................................................................... 14

Single Questions ................................................................................ 15

Decision Sets..................................................................................... 16

Clustered Questions ........................................................................... 22

Multiple-Choice Practice Questions ................................................................ 23

Answer Key and Rationales .......................................................................... 57

Study Plan Sheet ........................................................................................ 78

Preparation Resources ................................................................................. 79

NOTE: After clicking on a link, right click and select “Previous View” to go back to original text.

TExES Principal (068) 2


About The Test

Test Name Principal

Test Code 068

Time 5 hours

Number of Questions 120 multiple-choice questions

Format Computer-administered test (CAT)

The TExES Principal (068) test is designed to assess whether a test taker has the
requisite knowledge and skills that an entry-level principal or assistant principal in
Texas public schools must possess. The 120 multiple-choice questions are based on
the Principal test framework. Questions on this test range from grades EC–12. The
test may contain questions that do not count toward the score.

The test contains a set of questions pertaining to the Texas Academic Performance
Report (TAPR) with authentic materials.

The number of scored questions will not vary; however, the number of questions
that are not scored may vary in the actual test. Your final scaled score will be based
only on scored questions.

NOTE: After clicking on a link, right click and select “Previous View” to go back to original text.

TExES Principal (068) 3


The Domains

Approx.
Percentage
Domain Domain Title of Test Competencies Assessed

I. School Community Leadership 33% 001−003

II. Instructional Leadership 44% 004−007

III. Administrative Leadership 23% 008−009

NOTE: After clicking on a link, right click and select “Previous View” to go back to original text.

TExES Principal (068) 4


The Standards
Principal Standard I
Learner-Centered Values and Ethics of Leadership: A principal is an educational
leader who promotes the success of all students by acting with integrity and
fairness and in an ethical manner.

Principal Standard II
Learner-Centered Leadership and Campus Culture: A principal is an educational
leader who promotes the success of all students and shapes campus culture by
facilitating the development, articulation, implementation, and stewardship of a
vision of learning that is shared and supported by the school community.

Principal Standard III


Learner-Centered Human Resources Leadership and Management: A principal is an
educational leader who promotes the success of all students by implementing a
staff evaluation and development system to improve the performance of all staff
members, selects and implements appropriate models for supervision and staff
development, and applies the legal requirements for personnel management.

Principal Standard IV
Learner-Centered Communications and Community Relations: A principal is an
educational leader who promotes the success of all students by collaborating with
families and community members, responding to diverse community interests and
needs, and mobilizing community resources.

Principal Standard V
Learner-Centered Organizational Leadership and Management: A principal is an
educational leader who promotes the success of all students through leadership and
management of the organization, operations, and resources for a safe, efficient,
and effective learning environment.

Principal Standard VI
Learner-Centered Curriculum Planning and Development. A principal is an
educational leader who promotes the success of all students by facilitating the
design and implementation of curricula and strategic plans that enhance teaching
and learning; alignment of curriculum, curriculum resources, and assessment; and
the use of various forms of assessment to measure student performance.

Principal Standard VII


Learner-Centered Instructional Leadership and Management. A principal is an
educational leader who promotes the success of all students by advocating,
nurturing, and sustaining a campus culture and instructional program conducive to
student learning and staff professional growth.

NOTE: After clicking on a link, right click and select “Previous View” to go back to original text.

TExES Principal (068) 5


Domains and Competencies
The content covered by this test is organized into broad areas of content called
domains. Each domain covers one or more of the educator standards for this field.
Within each domain, the content is further defined by a set of competencies. Each
competency is composed of two major parts:

 The competency statement, which broadly defines what an entry-level


educator in this field in Texas public schools should know and be able to do.
 The descriptive statements, which describe in greater detail the knowledge
and skills eligible for testing.

Domain I — School Community Leadership

Competency 001: The principal knows how to shape campus culture by facilitating
the development, articulation, implementation and stewardship of a vision of
learning that is shared and supported by the school community*.

The principal knows how to:

A. Create a campus culture that sets high expectations, promotes learning and
provides intellectual stimulation for self, students and staff.
B. Ensure that parents and other members of the community are an integral
part of the campus culture.
C. Implement strategies to ensure the development of collegial relationships
and effective collaboration.
D. Respond appropriately to diverse needs in shaping the campus culture.
E. Use various types of information (e.g., demographic data, campus climate
inventory results, student achievement data, emerging issues affecting
education) to develop a campus vision and create a plan for implementing
the vision.
F. Use strategies for involving all stakeholders in planning processes to enable
the collaborative development of a shared campus vision focused on teaching
and learning.
G. Facilitate the collaborative development of a plan that clearly articulates
objectives and strategies for implementing a campus vision.
H. Align financial, human and material resources to support implementation of a
campus vision.
I. Establish procedures to assess and modify implementation plans to ensure
achievement of the campus vision.
J. Support innovative thinking and risk taking within the school community and
view unsuccessful experiences as learning opportunities.

NOTE: After clicking on a link, right click and select “Previous View” to go back to original text.

TExES Principal (068) 6


K. Acknowledge and celebrate the contributions of students, staff, parents and
community members toward realization of the campus vision.

* School Community includes students, staff, parents/caregivers and community


members.

Competency 002: The principal knows how to communicate and collaborate with all
members of the school community, respond to diverse interests and needs and
mobilize resources to promote student success.

The principal knows how to:

A. Communicate effectively with families and other community members in


varied educational contexts.
B. Apply skills for building consensus and managing conflict.
C. Implement effective strategies for systematically communicating with and
gathering input from all campus stakeholders.
D. Develop and implement strategies for effective internal and external
communications.
E. Develop and implement a comprehensive program of community relations
that effectively involves and informs multiple constituencies, including the
media.
F. Provide varied and meaningful opportunities for parents/caregivers to be
engaged in the education of their children.
G. Establish partnerships with parents/caregivers, businesses and others in the
community to strengthen programs and support campus goals.
H. Communicate and work effectively with diverse groups in the school
community to ensure that all students have an equal opportunity for
educational success.
I. Respond to pertinent political, social and economic issues in the internal and
external environment.

Competency 003: The principal knows how to act with integrity, fairness and in an
ethical and legal manner.

The principal knows how to:

A. Model and promote the highest standard of conduct, ethical principles and
integrity in decision making, actions and behaviors.
B. Implement policies and procedures that promote professional educator
compliance with The Code of Ethics and Standard Practices for Texas
Educators.

NOTE: After clicking on a link, right click and select “Previous View” to go back to original text.

TExES Principal (068) 7


C. Apply knowledge of ethical issues affecting education.
D. Apply legal guidelines (e.g., in relation to students with disabilities, bilingual
education, confidentiality, discrimination) to protect the rights of students
and staff and to improve learning opportunities.
E. Apply laws, policies and procedures in a fair and reasonable manner.
F. Articulate the importance of education in a free democratic society.
G. Serve as an advocate for all children.
H. Promote the continuous and appropriate development of all students.
I. Promote awareness of learning differences, multicultural awareness, gender
sensitivity and ethnic appreciation.

Domain II − Instructional Leadership

Competency 004: The principal knows how to facilitate the design and
implementation of curricula and strategic plans that enhance teaching and learning;
ensure alignment of curriculum, instruction, resources and assessment; and
promote the use of varied assessments to measure student performance.

The principal knows how to:

A. Facilitate effective campus curriculum planning based on knowledge of


various factors (e.g., emerging issues, occupational and economic trends,
demographic data, student learning data, motivation theory, teaching and
learning theory, principles of curriculum design, human developmental
processes, legal requirements).
B. Facilitate the use of sound, research-based practice in the development,
implementation and evaluation of campus curricular, co-curricular and
extracurricular programs.
C. Facilitate campus participation in collaborative district planning,
implementation, monitoring and revision of curriculum to ensure appropriate
scope, sequence, content and alignment.
D. Facilitate the use of appropriate assessments to measure student learning
and ensure educational accountability.
E. Facilitate the use of technology, telecommunications and information systems
to enrich the campus curriculum.
F. Facilitate the effective coordination of campus curricular, co-curricular and
extracurricular programs in relation to other district programs.
G. Promote the use of creative thinking, critical thinking and problem solving
by staff and other campus stakeholders involved in curriculum design
and delivery.

NOTE: After clicking on a link, right click and select “Previous View” to go back to original text.

TExES Principal (068) 8


Competency 005: The principal knows how to advocate, nurture and sustain an
instructional program and a campus culture that are conducive to student learning
and staff professional growth.

The principal knows how to:

A. Facilitate the development of a campus learning organization that supports


instructional improvement and change through ongoing study of relevant
research and best practice.
B. Facilitate the implementation of sound, research-based instructional
strategies, decisions and programs in which multiple opportunities to learn
and be successful are available to all students.
C. Create conditions that encourage staff, students, families/caregivers and the
community to strive to achieve the campus vision.
D. Ensure that all students are provided high-quality, flexible instructional
programs with appropriate resources and services to meet individual student
needs.
E. Use formative and summative student assessment data to develop, support
and improve campus instructional strategies and goals.
F. Facilitate the use and integration of technology, telecommunications and
information systems to enhance learning.
G. Facilitate the implementation of sound, research-based theories and
techniques of teaching, learning, classroom management, student discipline
and school safety to ensure a campus environment conducive to teaching
and learning.
H. Facilitate the development, implementation, evaluation and refinement of
student services and activity programs to fulfill academic, developmental,
social and cultural needs.
I. Analyze instructional needs and allocate resources effectively and equitably.
J. Analyze the implications of various factors (e.g., staffing patterns, class
scheduling formats, school organizational structures, student discipline
practices) for teaching and learning.
K. Ensure responsiveness to diverse sociological, linguistic, cultural and other
factors that may affect students’ development and learning.

NOTE: After clicking on a link, right click and select “Previous View” to go back to original text.

TExES Principal (068) 9


Competency 006: The principal knows how to implement a staff evaluation and
development system to improve the performance of all staff members, select and
implement appropriate models for supervision and staff development and apply the
legal requirements for personnel management.

The principal knows how to:

A. Work collaboratively with other campus personnel to develop, implement,


evaluate and revise a comprehensive campus professional development
plan that addresses staff needs and aligns professional development with
identified goals.
B. Facilitate the application of adult learning principles and motivation theory
to all campus professional development activities, including the use of
appropriate content, processes and contexts.
C. Allocate appropriate time, funding and other needed resources to ensure the
effective implementation of professional development plans.
D. Implement effective, appropriate and legal strategies for the recruitment,
screening, selection, assignment, induction, development, evaluation,
promotion, discipline and dismissal of campus staff.
E. Use formative and summative evaluation procedures to enhance the
knowledge and skills of campus staff.
F. Diagnose campus organizational health and morale and implement strategies
to provide ongoing support to campus staff.
G. Engage in ongoing professional development activities to enhance one’s own
knowledge and skills and to model lifelong learning.

NOTE: After clicking on a link, right click and select “Previous View” to go back to original text.

TExES Principal (068) 10


Competency 007: The principal knows how to apply organizational, decision-making
and problem-solving skills to ensure an effective learning environment.

The principal knows how to:

A. Implement appropriate management techniques and group process skills


to define roles, assign functions, delegate authority and determine
accountability for campus goal attainment.
B. Implement procedures for gathering, analyzing and using data from a variety
of sources for informed campus decision making.
C. Frame, analyze and resolve problems using appropriate problem-solving
techniques and decision-making skills.
D. Use strategies for promoting collaborative decision making and problem
solving, facilitating team building and developing consensus.
E. Encourage and facilitate positive change, enlist support for change and
overcome obstacles to change.
F. Apply skills for monitoring and evaluating change and making needed
adjustments to achieve goals.

Domain III — Administrative Leadership

Competency 008: The principal knows how to apply principles of effective


leadership and management in relation to campus budgeting, personnel, resource
utilization, financial management and technology use.

The principal knows how to:

A. Apply procedures for effective budget planning and management.


B. Work collaboratively with stakeholders to develop campus budgets.
C. Acquire, allocate and manage human, material and financial resources
according to district policies and campus priorities.
D. Apply laws and policies to ensure sound financial management in relation to
accounts, bidding, purchasing and grants.
E. Use effective planning, time management and organization of personnel to
maximize attainment of district and campus goals.
F. Develop and implement plans for using technology and information systems
to enhance school management.

NOTE: After clicking on a link, right click and select “Previous View” to go back to original text.

TExES Principal (068) 11


Competency 009: The principal knows how to apply principles of leadership and
management to the campus physical plant and support systems to ensure a safe
and effective learning environment.

The principal knows how to:

A. Implement strategies that enable the school physical plant, equipment and
support systems to operate safely, efficiently and effectively.
B. Apply strategies for ensuring the safety of students and personnel and for
addressing emergencies and security concerns.
C. Develop and implement procedures for crisis planning and for responding
to crises.
D. Apply local, state and federal laws and policies to support sound decision
making related to school programs and operations (e.g., student services,
food services, health services, transportation).

NOTE: After clicking on a link, right click and select “Previous View” to go back to original text.

TExES Principal (068) 12


Approaches to Answering Multiple-Choice Questions
The purpose of this section is to describe multiple-choice question formats that
you will typically see on the Principal test and to suggest possible ways to approach
thinking about and answering them. These approaches are intended to supplement
and complement familiar test-taking strategies with which you may already be
comfortable and that work for you. Fundamentally, the most important component
in assuring your success on the test is knowing the content described in the test
framework. This content has been carefully selected to align with the knowledge
required to begin a career as a Principal or assistant Principal.

The multiple-choice questions on this test are designed to assess your knowledge
of the content described in the test framework. In most cases, you are expected to
demonstrate more than just your ability to recall factual information. You may be
asked to think critically about a situation, to analyze it, consider it carefully,
compare it with other knowledge you have or make a judgment about it.

When you are ready to respond to a multiple-choice question, you must choose
one of four answer options. Leave no questions unanswered. Questions for which
you mark no answer or mark more than one answer are counted as incorrect. Your
score will be determined by the number of questions for which you select the
correct answer.

The Principal test is designed to include a total of 120 multiple-choice questions.


The number of scored questions will not vary; however, the number of questions
that are not scored may vary in the actual test. Your final scaled score will be based
only on scored questions. The questions that are not scored are being pilot tested
to collect information about how these questions will perform under actual testing
conditions. These pilot questions are not identified on the test.

How to Approach Unfamiliar Question Formats

Some questions include introductory information such as a map, table, graph or


reading passage (often called a stimulus) that provides the information the question
asks for. New formats for presenting information are developed from time to time.
Tests may include audio and video stimulus materials such as a movie clip or some
kind of animation, instead of a map or reading passage. Other tests may allow you
to zoom in on the details in a graphic or picture.

Tests may also include interactive types of questions. These questions take
advantage of technology to assess knowledge and skills that go beyond what can
be assessed using standard single-selection multiple-choice questions. If you see a
format you are not familiar with, read the directions carefully. The directions
always give clear instructions on how you are expected to respond.

NOTE: After clicking on a link, right click and select “Previous View” to go back to original text.

TExES Principal (068) 13


For most questions, you will respond by clicking an oval to choose a single answer
choice from a list of options. Other questions may ask you to respond by:

 Selecting all that apply. In some questions, you will be asked to choose all
the options that answer the question correctly.
 Typing in an entry box. When the answer is a number, you might be asked
to enter a numeric answer or, if the test has an on-screen calculator, you
might need to transfer the calculated result from the calculator into the entry
box. Some questions may have more than one place to enter a response.
 Clicking check boxes. You may be asked to click check boxes instead of an
oval when more than one choice within a set of answers can be selected.
 Clicking parts of a graphic. In some questions, you will choose your
answer by clicking on location(s) on a graphic such as a map or chart, as
opposed to choosing from a list.
 Clicking on sentences. In questions with reading passages, you may be
asked to choose your answer by clicking on a sentence or sentences within
the reading passage.
 Dragging and dropping answer choices into “targets” on the screen.
You may be asked to choose an answer from a list and drag it into the
appropriate location in a table, paragraph of text or graphic.
 Selecting options from a drop-down menu. This type of question will ask
you to select the appropriate answer or answers by selecting options from a
drop-down menu (e.g., to complete a sentence).
Remember that with every question, you will get clear instructions on how
to respond.

Question Formats

You may see the following types of multiple-choice questions on the test:

— Single Questions
— Decision Sets
— Clustered Questions

On the following pages, you will find descriptions of these commonly used question
formats, along with some suggested approaches for responding to them.

NOTE: After clicking on a link, right click and select “Previous View” to go back to original text.

TExES Principal (068) 14


Single Questions

The single-question format presents a direct question or an incomplete statement.


It can also include a reading passage, graphic, table or a combination of these. Four
or more answer options appear below the question.

The following question is an example of the single-question format. It tests


knowledge of Principal Competency 004: The principal knows how to facilitate the
design and implementation of curricula and strategic plans that enhance teaching
and learning; ensure alignment of curriculum, instruction, resources and
assessment; and promote the use of varied assessments to measure student
performance.

Example

One goal of a campus curricular program is to produce strong student


performance as demonstrated by standardized measures. Which of the
following factors is likely to be the most accurate predictor of the program’s
effectiveness in meeting this goal??

A. The degree of consistency across subject areas and grade levels of the
instructional approaches used to present content
B. The percent of the total campus budget that is devoted to supporting
programs related to instruction in the core curriculum
C. The extent of alignment among the content identified in curriculum
documents, the content actually delivered and the content assessed
D. The percent of the total campus budget that is devoted to supporting
innovative curricular programs such as the instructional use of technology

Suggested Approach

Read the question carefully and critically. Think about what it is asking and the
situation it is describing. Eliminate any obviously wrong answers, select the correct
answer choice and mark your answer.

For example, as you read this question, consider the specified goal: producing
strong student performance as demonstrated by standardized measures. The one
factor that would be critical to achieving this goal would be the alignment of
curriculum, instruction and assessment. If, for example, the tests used did not
measure what was actually taught in the classroom, students would be unlikely to
perform well on the tests. Thus, such alignment would be an important predictor of
effectiveness in meeting the stated goal (option C). Whether teachers campus wide
use the same instructional approaches (option A) would not be a significant factor;
the same content can be taught effectively in many different ways.

NOTE: After clicking on a link, right click and select “Previous View” to go back to original text.

TExES Principal (068) 15


With regard to the other two options, the percentage of the campus budget
allocated either to programs supporting the core curriculum (option B) or to
innovative programs (option D) would be largely irrelevant to student performance
on standardized measures if curriculum, instruction and assessment were not
aligned. Therefore, option C is the correct answer.

Decision Sets

Decision sets contain approximately five to nine questions that are all about a
particular situation faced by a principal in a hypothetical Texas school. The decision-
set format helps to simulate situations that school principals encounter and in which
they must make decisions that affect staff, students and the larger school
community. Each decision set begins with stimulus material, which sets the scene
and provides information to use in answering the questions that follow. Decision
sets also contain additional stimulus material provided at one or more junctures.
This stimulus material typically provides additional information or a subsequent
development that you will need to answer the questions that follow.

The decision-set stimulus material may include demographic information about the
district, a description of the community served by the district or information about
district staff in general or one or more particular staff members. Each decision-set
stimulus describes a situation that a principal is facing.

The beginning and end of each decision set will be clearly marked. The following
sample shows the first stimulus for a decision set about a middle school principal,
Ms. James.

Decision Set: First Stimulus

DECISION SET BEGINS HERE

Part 1

Ms. James, the new principal of Harmon Middle School, reviews the results of
the annual state assessment. She finds a wide discrepancy in the achievement
levels of different ethnic subgroups identified in the assessment results on the
campus. She decides to investigate further by interviewing students, parents
and teachers and evaluating the campus climate.

The stimulus material presents essential information for the decision set. This
stimulus indicates that Ms. James has recently become the principal of Harmon
Middle School and that she has detected an important “red flag” in students’ test
data: a gap in achievement levels among different groups of students. The stimulus
also indicates that Ms. James does not plan to take any immediate action but will
first take steps to find out more about the situation.

NOTE: After clicking on a link, right click and select “Previous View” to go back to original text.

TExES Principal (068) 16


In a decision set, the stimulus is an important scene setter. As you consider the
test questions that follow, check the stimulus as needed. As you progress through
the decision set, additional stimuli may introduce new information. The decision set
is often set up as a kind of story line that develops over time. You may find it
helpful to skim all of the questions and stimulus materials in a decision set before
answering any questions. Such a quick read-through may give you a richer
understanding of the context for the questions. However, as you select your answer
for each question, be sure to base your choice on only the information that has
already been presented about the situation.
Regardless of the strategy you use, keep in mind that every piece of information in
each stimulus and question is important. You should base your answers on the
particular situation presented, not on similar situations you have encountered or
studied. With each question, you should think about the knowledge and skills
outlined in the test framework.

Decision Set

Each stimulus can be followed by one or more questions. This stimulus is followed
by one question. It assesses knowledge of Principal Competency 005: The principal
knows how to advocate, nurture and sustain an instructional program and a campus
culture that are conducive to student learning and staff professional growth.

Example 1

To better understand the learning environment at Harmon, Ms. James observes


students in a variety of school settings. The clearest indication that the needs of
all students are not being met is the observation that particular subgroups of
students

A. are socializing primarily with peers from their cultural background.


B. sometimes require modifications or support during instruction.
C. are adopting behaviors of schoolmates from other cultural groups.
D. exhibit lack of confidence and motivation regarding schoolwork.

Suggested Approach

Read the question carefully and critically. Think about what it is asking and the
situation it is describing. Eliminate any obviously wrong answers, select the correct
answer choice and mark your answer.

For example, as you read this question, think about the factors a principal must
consider in assessing whether all members of a learning community have an
equitable opportunity to achieve. As she observes students in her school, Ms. James
must distinguish student behavior patterns that indicate normal preadolescent
development from patterns that may indicate that the needs of some students are
not being met. In a middle school setting, peer groups are particularly important in
students’ development. The fact that students of a particular ethnic background

NOTE: After clicking on a link, right click and select “Previous View” to go back to original text.

TExES Principal (068) 17


tend to socialize together is natural; peer groups typically form because their
members have something important in common (option A). It is expected that
some students may require additional assistance or attention from teachers
(option B). Also, students seeking to identify with a particular ethnic group may
well adopt some of that group’s behaviors (option C). If, however, students from
particular groups consistently show less confidence and motivation than other
students, the principal should recognize that lowered expectations for those
students might be the reason and that she should actively work to ensure that any
such attitude is eliminated. Therefore, option D is the correct response.

Decision Set: Second Stimulus

The next four sample questions in this section are a part of the same decision set.
These four questions follow a second stimulus. Each sample question is followed by
a suggested approach to answering it. Additional stimulus material is sometimes
provided in a decision set to provide more information or to introduce a new
development in a situation. In this case, the following stimulus supplies the
conclusion Ms. James reaches after interviewing faculty members.

Part 2

After talking with numerous faculty members, Ms. James concludes that many
teachers are unsure about what they ought to be doing to meet the needs of the
campus’s diverse population. She decides to bring up this issue at a faculty
meeting.

Example 2

The following sample question tests knowledge of Principal Competency 005: The
principal knows how to advocate, nurture and sustain an instructional program and
a campus culture that are conducive to student learning and staff professional
growth.

At one point in the meeting, many faculty members say they want to find
practical ways to foster a strong sense of community among students in their
classrooms. Which of the following is the best instructional strategy for
Ms. James to suggest?

A. Regularly calling on students to present their ethnic perspective on issues


being studied in class
B. Developing a series of mini-units, each designed to highlight and celebrate
one ethnic group represented in the classroom
C. Regularly implementing classroom activities that involve small, ethnically
mixed groups of students
D. Involving all students in a project to explore relationships among different
ethnic groups in their local community

NOTE: After clicking on a link, right click and select “Previous View” to go back to original text.

TExES Principal (068) 18


Suggested Approach

To answer this question, you need to have an understanding of the kinds of


activities that will help to build appreciation for diversity, an awareness of common
needs and the ability to work collaboratively toward common ends. Options A and B
might result in students’ viewing their own or others’ groups as singled out or
celebrated only at certain times. Involving all students in exploring relationships
among ethnic groups in the community (option D) may be a worthy project but
could focus students’ attention outside the school community. Building a sense of
community among students, especially in a middle school setting, can best be
facilitated if the students have regular, meaningful opportunities to collaborate on
classroom activities. As students work together, they will discover similarities and
shared values that may enhance their collaboration Option C is the correct
response.

Example 3

The following sample question tests knowledge of Principal Competency 002: The
principal knows how to communicate and collaborate with all members of the school
community, respond to diverse interests and needs and mobilize resources to
promote student success.

Ms. James suggests that all students would have a better chance to succeed if
teachers developed effective communication with the students’ families. The
teachers agree, but they point out that some families seem reluctant to talk to
them. Which of the following steps should Ms. James suggest that teachers try
first to facilitate communication with these families?

A. Telephoning each family to ask how the teacher can help make school/home
communication more comfortable for them
B. Posting a letter on the school website explaining the importance of frequent
interaction and communication between teachers and families
C. Examining their own interactions with families to determine whether they as
teachers have been sensitive to cultural and language differences
D. Asking school counselors to meet with students from various cultural
backgrounds to discover better ways to approach the students’ families

Suggested Approach

This question focuses on a principal’s responsibility for the flow of communication to


various groups that make up the learning community and for ensuring that school
staff are sensitive to the needs of those groups. Telephoning families to address the
issue directly (option A) is an approach that some families may find intrusive or
embarrassing. Posting a letter to families (option B) is impersonal and sets the
school up as authority rather than partner. Option D asks the counselors to take on
a problem that needs to be addressed by the entire school community. Since all
teachers communicate regularly with the families of the students in their classes, it

NOTE: After clicking on a link, right click and select “Previous View” to go back to original text.

TExES Principal (068) 19


is appropriate for the principal to suggest that teachers should first examine their
prior communication efforts to determine which have been conducive to effective
communication. Option C is the correct response.

Example 4

The following sample question tests knowledge of Principal Competency 001: The
principal knows how to shape campus culture by facilitating the development,
articulation, implementation and stewardship of a vision of learning that is shared
and supported by the school community.

Ms. James is concerned that a few teachers may “write off” some students each
year. Which of the following should be Ms. James’s first step in motivating
these faculty members to begin addressing the needs of all students?

A. Asking faculty to submit ideas for staff development that could enhance
teachers’ abilities to address all students’ needs
B. Distributing to faculty legal and ethical guidelines about educational equity
for all students
C. Providing faculty with easy-to-implement strategies that could immediately
improve instruction for all students
D. Encouraging faculty discussion that will lead to a campus-wide commitment
to hold high standards for all students

Suggested Approach

This question requires a knowledge of strategies that a principal can use to guide
the development of a vision that reflects commitment to all students’ academic
achievement and success in life. Before any specific strategies are implemented,
the faculty must have an understood, shared commitment to holding high
standards for the needs of all students, which makes option D the correct
response. Encouraging faculty to submit ideas about staff development activities
(option A) is a good strategy, but it will not work without a shared commitment.
Distributing legal and ethical guidelines (option B) before the faculty comes to a
consensus might actually cause resentment, thereby exacerbating the problem.
Providing the faculty with easy-to-implement strategies (option C) might give the
impression that the problem can be solved easily or that solving it is optional.
Working for a campus-wide commitment, the best option, shows strong leadership
and is likely to lead to positive long-term change.

Example 5

The following sample question tests knowledge of Principal Competency 002: The
principal knows how to communicate and collaborate with all members of the school
community, respond to diverse interests and needs and mobilize resources to
promote student success.

NOTE: After clicking on a link, right click and select “Previous View” to go back to original text.

TExES Principal (068) 20


An article in the local paper about Harmon’s intention to improve education for
all students leads to public disagreement and animosity among constituencies
who tend to have widely differing views about problems on campus and how to
address them. Which of the following strategies used by Ms. James and her
staff will best encourage more productive dialogue and a more positive attitude
among these groups?

A. Inviting stakeholders with various viewpoints to meet and reach consensus


on ways to address areas of disagreement
B. Disseminating to key members of each constituency important information
about major areas of disagreement
C. Establishing a clearly defined set of procedures for resolving disagreements
D. Advising all constituencies that the students are the ones most likely to be
hurt by their disagreements

Suggested Approach

This question centers on the school’s role in reaching out to include the members of
the larger community as partners with the learning community. Simply
disseminating information about major areas of disagreement (option B) is not
likely to move constituency groups closer to agreement.
Establishing a set of procedures for resolving disagreements (option C) could
communicate an unwillingness on the part of the school leader to invite open
discussion. Advising some members of the learning community that their actions
might hurt children (option D) is likely to create more animosity; the principal
should keep in mind that most constituency groups’ views are based on their own
interests in children’s education. The principal is responsible for communicating with
various groups in the community and must also listen and encourage honest
dialogue among groups, helping them to understand the learning community’s
vision and including them in the decision-making process. Inviting involvement of
people representing different points of view will likely win community support.
Option A is the correct response.

DECISION SET ENDS HERE

NOTE: After clicking on a link, right click and select “Previous View” to go back to original text.

TExES Principal (068) 21


Clustered Questions

Clustered questions are made up of a stimulus and two or more questions relating
to the stimulus. The stimulus material can be a reading passage, description of an
experiment, graphic, table or any other information necessary to answer the
questions that follow.

You can use several different approaches to respond to clustered questions. Some
commonly used strategies are listed below.

Strategy 1 Skim the stimulus material to understand its purpose, its


arrangement and/or its content. Then read the questions and refer
again to the stimulus material to obtain the specific information you
need to answer the questions.

Strategy 2 Read the questions before considering the stimulus material. The
theory behind this strategy is that the content of the questions will
help you identify the purpose of the stimulus material and locate the
information you need to answer the questions.

Strategy 3 Use a combination of both strategies. Apply the “read the stimulus
first” strategy with shorter, more familiar stimuli and the “read the
questions first” strategy with longer, more complex or less familiar
stimuli. You can experiment with the sample questions in this manual
and then use the strategy with which you are most comfortable when
you take the actual test.

Whether you read the stimulus before or after you read the questions, you should
read it carefully and critically. You may want to note its important points to help
you answer the questions.

As you consider questions set in educational contexts, try to enter into the
identified superintendent’s frame of mind and use that superintendent’s point of
view to answer the questions that accompany the stimulus. Be sure to consider the
questions only in terms of the information provided in the stimulus — not in terms
of your own experiences or individuals you may have known.

NOTE: After clicking on a link, right click and select “Previous View” to go back to original text.

TExES Principal (068) 22


Multiple-Choice Practice Questions
This section presents some sample test questions for you to review as part of your
preparation for the test. To demonstrate how each competency may be assessed,
each sample question is accompanied by the competency that it measures. While
studying, you may wish to read the competency before and after you consider each
sample question. Please note that the competency statements do not appear on the
actual test.

For each sample test question, there is a correct answer and a rationale for each
answer option. Please note that the sample questions are not necessarily presented
in competency order.

The sample questions are included to illustrate the formats and types of questions
you will see on the test; however, your performance on the sample questions
should not be viewed as a predictor of your performance on the actual test.

NOTE: After clicking on a link, right click and select “Previous View” to go back to original text.

TExES Principal (068) 23


Questions 1–3 refer to the following information.

Ms. Manning, the new principal at Starling Middle School, joins the staff midyear.
She learns that Ms. Jones, a first-year teacher, is struggling. She also reviews data
showing that the scores on the science benchmark assessments are low in Ms.
Jones’s classroom. Ms. Manning realizes that it will take time to fully assess campus
needs but sees an urgency in reviewing Ms. Jones’s performance and addressing
the low benchmark scores.

COMPETENCY 006

1. The assistant principal evaluating Ms. Jones provides Ms. Manning with all
evaluation documentation, which includes classroom observations, a report
from her mentor, and recommendations for improvement. Which of the
following questions is most important for Ms. Manning to answer when she
considers renewing Ms. Jones’s contract?

A. Does Ms. Jones feel that there is more the campus can do to support her?
B. Does Ms. Jones meet the legal requirements for termination? 

C. Did Ms. Jones receive new-employee training from the district office?
D. To what degree has Ms. Jones adjusted to her new role as teacher?

Answer and Rationale

COMPETENCY 007

2. In the next weeks, Ms. Manning examines the science test scores, report card
grades, and benchmark assessments of all students. She identifies a number of
concerns and concludes that the science teachers will benefit from reviewing
the data. She plans to hold meetings with the science teachers and the
department coordinator. Her goal is to share what she has learned, gather
feedback, and develop an action plan. In doing so, Ms. Manning is

A. identifying ways that student diversity should be addressed in the


classroom.
B. implementing a plan that accounts for what motivates adults and how they
learn best. 

C. engaging in ongoing professional development activities to enhance
knowledge. 

D. analyzing data from a variety of sources and managing groups to attain
campus goals.

Answer and Rationale

NOTE: After clicking on a link, right click and select “Previous View” to go back to original text.

TExES Principal (068) 24


COMPETENCY 001

3. During one of the meetings, Ms. Manning overhears science teachers


commenting that they think she is too new to the campus to be making so
many changes. The teachers express that she does not know them, their
students, or the community well. Based on this information, what is Ms.
Manning’s biggest obstacle in creating change?

A. Developing a vision understood by the campus 



B. Becoming legitimate in the teachers’ eyes 

C. Providing a general sense of direction 

D. Overcoming unforeseen barriers

Answer and Rationale

Questions 4–6 refer to the following information.

Mr. Ryoko has just been hired as the new principal of Greenville Elementary School.
He wants to begin his tenure as principal on a positive note. To accomplish this, he
develops the following plan.

STEPS

1. Hold meetings with faculty at the beginning of the year for teachers to share
their views on campus strengths, needs, and strategies for improvement. 

2. Work with teachers to develop a campus mission statement and key goals to
fulfill the mission. 

3. Facilitate collaboration among teachers to review the curriculum, pedagogy,
and assessments to develop a specific plan to achieve the goals agreed upon
in step 2.

COMPETENCY 001

4. The primary benefit of the meetings described in step 1 is most likely to

A. give Mr. Ryoko a chance to define his expectations for the campus staff. 

B. acquaint the campus staff with Mr. Ryoko’s leadership style. 

C. provide Mr. Ryoko with a source of information for evaluating campus
needs. 

D. prompt the teachers to reflect on their current professional knowledge.

Answer and Rationale

NOTE: After clicking on a link, right click and select “Previous View” to go back to original text.

TExES Principal (068) 25


COMPETENCY 007

5. The greatest benefit of step 3 is that it is likely to

A. ensure innovation and creative problem solving to achieve educational


excellence. 

B. facilitate the identification of faculty members to spearhead the
implementation of specific action plans. 

C. prompt staff to define and use criteria to self-assess their performance. 

D. allow correlation of student learning with the use of specific teaching
practices.

Answer and Rationale

COMPETENCY 002

6. Mr. Ryoko’s plan could be best improved by

A. clarifying his own desired outcomes for the proposed changes.


B. delegating greater authority and control to the faculty.
C. modeling the mission statement after those of other district campuses.
D. involving parents, students, and other staff in the process.

Answer and Rationale

Questions 7–11 refer to the following information.

Decision Set Begins Here

PART 1

Ms. Singh, the principal of Groveville Elementary School, meets with teachers to
discuss the students’ mathematics scores on the most recent state-mandated
assessment. Although the scores show that the majority of students performed
satisfactorily, several teachers express that the mathematics curriculum does not
reflect current research on best practices in mathematics instruction. A similar
sentiment is expressed by the parent council. Some teachers are reluctant to revise
a curriculum that is producing satisfactory results, recalling that the last time
changes were made to the mathematics curriculum, test scores dropped and
protests from parents and the community resulted. Ms. Singh forms a committee of
teachers to investigate revisions to the current curriculum that will further develop
and enhance the students’ mathematics skills.

NOTE: After clicking on a link, right click and select “Previous View” to go back to original text.

TExES Principal (068) 26


COMPETENCY 001

7. Which of the following should Ms. Singh emphasize most as the committee
begins its work?

A. Establishing norms within the committee that will define its focus and
facilitate collaboration 

B. Providing educational data acquired from reputable sources to committee
members

C. Celebrating the contributions of students, staff, parents and community 

D. Communicating information to all members of the school and greater
communities

Answer and Rationale

COMPETENCY 004

8. Ms. Singh and the mathematics committee develop a set of questions as they
consider possible curriculum revisions. Which of the following questions is most
important to include?

A. Is the revision based on identified student strengths and needs? 



B. Is there an expense associated with implementing the revision? 

C. Is the revision compatible with the teaching style of the faculty members? 

D. Is the revision supported by the instructional resources the campus is
already using?

Answer and Rationale

NOTE: After clicking on a link, right click and select “Previous View” to go back to original text.

TExES Principal (068) 27


PART 2

After several months, the committee presents its recommended revisions.


Ms. Singh works with committee members to design a communication plan to
introduce and build support for the revisions. As part of the plan, Ms. Singh will
hold a meeting with parents and community members to discuss the proposed
revisions and the rationales behind them.

COMPETENCY 007

9. Which of the following is Ms. Singh’s best strategy for laying the groundwork for
long-term support from families and community members?

A. Communicating that it may take time for favorable changes in student


achievement to become evident after curriculum revision
B. Explaining the hard work, dedication, and focus that campus teachers have
put into the changes and requesting the community’s support 

C. Asking the community to trust the teachers’ judgment in identifying areas of
the curriculum requiring revision

D. Inviting community members to submit suggestions for additional revisions
after the curriculum is presented to the board of trustees

Answer and Rationale

COMPETENCY 002

10. Halfway through the meeting, several parents object to the proposed revisions,
stating that they see no reason to make changes. Ms. Singh could have
minimized the likelihood of such opposition by using which of the following
strategies to begin the meeting?

A. Describing the success of other elementary campuses in implementing new


approaches to mathematics instruction 

B. Acknowledging that many parents may have misgivings about changes in
the mathematics curriculum 

C. Presenting information about the data the committee used to identify areas
for revision and how the proposed revisions relate to those areas 

D. Explaining that parents would have an opportunity to comment on the
revisions after the presentation

Answer and Rationale

NOTE: After clicking on a link, right click and select “Previous View” to go back to original text.

TExES Principal (068) 28


COMPETENCY 002

11. Ms. Singh can best help ensure the success of the new curriculum by asking
parents to

A. contact her or any of the curriculum committee members to share their


thoughts and questions about the new curriculum. 

B. educate themselves about trends in mathematics education so they can be
informed partners in implementing the new curriculum. 

C. assess the effectiveness of the new curriculum by evaluating their children’s
mathematical skills. 

D. ask their children about daily assignments, monitor their work to make sure
it is completed, and praise their efforts.

Answer and Rationale

Decision Set Ends Here

Questions 12–15 refer to the following information.

Mr. Carruthers, a seventh-grade team leader, goes to Ms. Winston, the principal at
Raleigh Middle School, to discuss a matter of concern voiced during a team
meeting. He says teachers believe that they are not prepared to provide
appropriate instruction for students in special education who are placed in their
general education classrooms. The teachers further question why the special
education teachers are not providing instruction for these students.

COMPETENCY 006

12. Ms. Winston believes that the larger issues raised by Mr. Carruthers must be
addressed if students in special education are to be served appropriately. Her
discussion with Mr. Carruthers suggests that the school is in need of stronger
leadership in the area of

A. establishing formal communication channels through which teachers’


concerns can be addressed. 

B. preparing and supporting teachers to instruct students with specialized
needs. 

C. providing increased opportunities for teachers to participate in school
governance. 

D. identifying students who are in genuine need of special education services
more accurately.

Answer and Rationale

NOTE: After clicking on a link, right click and select “Previous View” to go back to original text.

TExES Principal (068) 29


COMPETENCY 003

13. Mr. Carruthers makes Ms. Winston aware of a student with a learning disability
who receives special education services. Despite interventions, the student
continues to struggle in mathematics. Mr. Carruthers asks Ms. Winston if he can
recommend that the student be transferred to a resource mathematics class.
Which of the following is Ms. Winston’s best response to Mr. Carruthers?

A. The placement of a student who receives special education services may


only be changed by the Admission Review Dismissal (ARD) committee. 

B. The director of special education must approve any change in a student’s
placement. 

C. Only a special education teacher can initiate an assessment that may result
in a special education student’s change in placement. 

D. Mr. Carruthers must document the reasons for his request before Ms.
Winston can change the student’s placement.

Answer and Rationale

COMPETENCY 003

14. Which of the following is the best initial step for Ms. Winston to take to ensure
that the needs of students in special education are being met?

A. Referring teachers to relevant resources available in the school library’s


professional collection 

B. Confirming that teachers are using students’ Individualized Education
Programs (IEPs) to identify the services they must provide 

C. Providing professional development on laws that govern the education of
students with disabilities 

D. Resolving to increase the amount of funding for special education services in
the next year’s budget

Answer and Rationale

NOTE: After clicking on a link, right click and select “Previous View” to go back to original text.

TExES Principal (068) 30


COMPETENCY 006

15. Ms. Winston receives an email from parents complaining that their child’s
teachers are not administering tests orally, as required by the student’s
Individualized Education Program (IEP). Ms. Winston’s primary responsibility in
regard to the complaint is to

A. explain to the parents why it is not always possible for classroom teachers
to adhere to an IEP. 

B. give teachers the opportunity to voice their objections pertaining to the
accommodations specified in the student’s IEP. 

C. forward the parents’ e-mail to the student’s teachers with a note asking
them to contact the parents. 

D. investigate the allegation and, if accurate, bring teacher practice in line with
the student’s IEP.

Answer and Rationale

Questions 16–20 refer to the following information.

Ms. Bailiff, the principal at Grover Elementary, reviews the school’s most recent
Texas Academic Performance Report (TAPR). Although the school met standards
according to the state’s rating system, she observes that student achievement
has decreased from the previous year. She plans a meeting with the campus
improvement team to analyze the data, determine the areas of greatest need, and
formulate an improvement plan.

The following questions will guide the data discussion.

1. Are we providing equal opportunities for all students to succeed?


2. What variables are affecting student achievement?
3. What should be the main focus of professional development for teachers?
4. Should we be concerned about our campus retention rate?

NOTE: The sample TAPR provided is intended to be representative only. The sample
report does not necessarily reflect the current reporting formats.

NOTE: After clicking on a link, right click and select “Previous View” to go back to original text.

TExES Principal (068) 31


TExES Principal (068) 32
TExES Principal (068) 33
TExES Principal (068) 34
TExES Principal (068) 35
TExES Principal (068) 36
TExES Principal (068) 37
TExES Principal (068) 38
TExES Principal (068) 39
TExES Principal (068) 40
TExES Principal (068) 41
TExES Principal (068) 42
TExES Principal (068) 43
TExES Principal (068) 44
TExES Principal (068) 45
COMPETENCY 007

16. Based on the report, which of the following data provides the best answer to
question 1?

A. The percentage of students receiving special education or gifted services


B. The disparity between the mathematics scores of males and females
C. The score differences between White students and other identified student
subgroups
D. The number of experienced teachers assigned to remedial classes

Answer and Rationale

COMPETENCY 005

17. Based on the data in the report, which of the following variables should the
committee conclude is exerting the most influence on the academic
performance of the students at Grover?

A. Absentee rate
B. Socioeconomic status
C. Teacher level of experience
D. Student ethnicity

Answer and Rationale

COMPETENCY 005

18. Which of the following training topics is the committee’s most appropriate
recommendation for question 3?

A. Integrating blended learning to develop and enhance students’


postsecondary readiness
B. Using formative assessment to monitor student understanding and adjust
instruction
C. Applying nontraditional criteria to identify giftedness in young children
D. Meeting the academic and social needs of highly mobile students

Answer and Rationale

NOTE: After clicking on a link, right click and select “Previous View” to go back to original text.

TExES Principal (068) 46


COMPETENCY 007

19. Which of the following questions should the team address to adequately answer
question 4?

A. Does large class size increase retention rates?


B. Are we applying social promotion to special education students?
C. Are retention criteria being applied equally and consistently?
D. Are some teachers using instructional strategies that result in more student
retentions?

Answer and Rationale

COMPETENCY 006

20. The addition of which of the following personnel is likely to be most helpful for
achieving the goal in question 1?

A. African American and Hispanic teachers


B. Paraprofessionals to support English-language learners
C. A campus disciplinary officer
D. Additional special education teachers

Answer and Rationale

COMPETENCY 001

21. A middle school principal focuses campus professional development on ways to


communicate high expectations to all students, regardless of gender, ethnicity,
or disability. This topic is most effective for fulfilling which of the following
responsibilities of the principal?

A. Promoting the development of collegial relationships and teamwork among


diverse staff members 

B. Establishing a collaborative process for developing a shared vision of the
school’s mission 

C. Ensuring that the campus is in compliance with all state and federal
regulations 

D. Helping shape a campus culture that responds to the diverse needs of the
school community

Answer and Rationale

NOTE: After clicking on a link, right click and select “Previous View” to go back to original text.

TExES Principal (068) 47


COMPETENCY 001

22. Campus stakeholders want to develop a new vision of learning for an


elementary school. Which of the following strategies is most effective for the
stakeholders to pursue first when developing the new campus vision?

A. Modeling the new vision after the visions of other district campuses
B. Gathering information about campus and community values 

C. Aligning financial, human, and material resources behind the vision
D. Developing a collaborative plan to implement the new vision

Answer and Rationale

COMPETENCY 002

23. An elementary school principal is leading an initiative to improve some aspects


of campus operations based on campus goals. Throughout the process, the
principal has used a variety of strategies, such as establishing an advisory
committee, surveying parents and teachers for their input, and holding public
meetings to answer questions on the proposed changes. The principal is using
the strategies primarily to

A. promote a sense of ownership for any changes that will ultimately be


implemented. 

B. ensure that stakeholders respect each other’s perspectives. 

C. eliminate disagreements among stakeholders about the changes. 

D. reflect the will of the majority of the community on whether any change is
necessary.

Answer and Rationale

NOTE: After clicking on a link, right click and select “Previous View” to go back to original text.

TExES Principal (068) 48


COMPETENCY 002

24. The principal and the site-based decision-making (SBDM) committee at a


middle school have been involved in selecting a new curriculum for health
education. As the committee concludes its work and prepares to recommend a
curriculum, some of the team members express that they are now dissatisfied
with some aspects of the chosen curriculum and cannot recommend it as is.
Which of the following actions by the principal will best help the team move
forward?

A. Suggesting that the committee explore different curricula and select from
among them 

B. Asking the team to reexamine the aspects of the curriculum that are causing
dissatisfaction 

C. Reminding the team that going backward in their process will possibly delay
their milestone dates
D. Meeting with team members individually to gauge their position on the
recommended curriculum

Answer and Rationale

COMPETENCY 003

25. The parents of a student who receives special education services complain that
their son was not selected for the high school basketball team because he has a
disability. They demand that he be put on the team, and if he is not, they will
file a complaint with the Office for Civil Rights. Which of the following is the
principal’s most appropriate initial response?

A. Allowing the student to try out for the team again with his parents present
B. Directing the parents to the basketball coach for an assessment of their
son’s basketball skills
C. Contacting the district’s Title IX coordinator for advice on legal aspects of
the situation
D. Explaining the laws that relate to students with disabilities and the selection
criteria for competitive programs

Answer and Rationale

NOTE: After clicking on a link, right click and select “Previous View” to go back to original text.

TExES Principal (068) 49


COMPETENCY 004

26. While observing a mathematics lesson, a high school principal notes that the
teacher uses an online simulation focused on changing geometric shapes.
Students generate hypotheses about the shapes and then discuss them with a
partner in an online forum. The teacher’s primary purpose for using an online
simulation is to

A. reinforce routines and processes.


B. build workplace readiness.
C. develop critical-thinking skills.
D. maintain student interest.

Answer and Rationale

COMPETENCY 005

27. A high school junior and his parents have submitted a written request to the
principal asking that a creative writing course be added as an English
department elective in the coming school year. The student is an avid writer
who hopes to pursue a career in writing. Which of the following is the most
appropriate action for the principal to take in response to such a request?

A. Explaining to the parents that courses of study are designed to meet the
general interests of all students, not the special interests of one student

B. Asking members of the English department whether any of them are
qualified and willing to teach a creative writing course 

C. Presenting the parents’ request at a closed meeting of the board of trustees

D. Asking the curriculum committee to explore whether the level of student
interest would make such a course feasible

Answer and Rationale

NOTE: After clicking on a link, right click and select “Previous View” to go back to original text.

TExES Principal (068) 50


COMPETENCY 005

28. The new principal of an elementary school that has a record of low student
achievement believes that the school’s instructional program must be improved
significantly to make academic success a reality for all students. Which of
the following is the most appropriate first step for the principal to take to
encourage the teaching staff to engage in a process of instructional
improvement?

A. Meeting informally with individual teachers and remind them of the


importance of participating in efforts to improve instruction at the school 

B. Arranging to provide a stipend or other financial incentives to teachers who
plan and implement changes in their instructional programs 

C. Analyzing student performance data with teachers to help them identify
instructional areas that should be modified 

D. Asking teachers to monitor campus goals for instructional improvement and
the criteria for evaluating whether the goals have been achieved

Answer and Rationale

COMPETENCY 005

29. The student population at a school is becoming increasingly diverse with


respect to factors such as students’ home languages and cultural backgrounds.
Which of the following responses to the school’s changing demographics should
the school leadership institute first?

A. Instructional groupings that place students with commonalities together 



B. A unit of instruction that focuses on multicultural themes
C. An assessment of the capability of the current support systems to meet the
needs of the new students
D. Exemptions from challenging courses to allow students to have successful
academic experiences

Answer and Rationale

NOTE: After clicking on a link, right click and select “Previous View” to go back to original text.

TExES Principal (068) 51


COMPETENCY 005

30. Campus teachers undergo professional development on implementing a


mastery learning model to improve student achievement. Which of the
following observations by the principal is the best evidence that teachers are
successfully applying the model?

A. Teachers collaborate on a regular basis to plan lessons for students who do


not meet curriculum standards on district and state assessments.
B. Department chairs facilitate teacher discussion during weekly lesson
planning sessions, encouraging them to maintain rigor in classroom
assessments.
C. Formative assessments are given during a study unit to measure student
understanding so that reteaching and retesting can occur immediately if
needed.
D. Students who do not pass summative assessments receive individualized
instructional support from paraprofessionals in the classroom.

Answer and Rationale

COMPETENCY 006

31. Which THREE of the following practices by a school leader are most effective for
building staff morale?

A. Striving to provide resources requested by teachers


B. Recognizing teachers publicly for exemplary performance
C. Relieving teachers of the pressure of making leadership decisions
D. Involving teachers in planning and evaluating schoolwide initiatives
E. Promoting competition among teachers to motivate high performance

Answer and Rationale

NOTE: After clicking on a link, right click and select “Previous View” to go back to original text.

TExES Principal (068) 52


COMPETENCY 007

32. Educators at a middle school will soon implement a new language arts
curriculum. Which of the following is the most effective method for obtaining an
objective assessment of the success of the new curriculum?

A. Developing a plan for gathering data on student performance at selected


points before and during implementation 

B. Reviewing students’ report card grades at the end of the first marking term
for evidence of improvement 

C. Comparing the scores of students on the previous year’s state-mandated
assessment with those of students in the current year
D. Surveying representative students and parents for their impressions of the
effectiveness of the new program

Answer and Rationale

COMPETENCY 007

33. A high school’s site-based decision-making (SBDM) committee is considering an


open-campus policy that allows students to eat lunch off campus. Based on
research on open-campus plans, the committee develops four options ranging
from the current closed-campus policy to an unrestricted open-campus policy.
Which of the following is the committee’s best next step in the decision-making
process?

A. Selecting one option by majority vote and building support for the option
within the school community 

B. Holding assemblies for the students to voice support for the options they
prefer 

C. Conducting a pilot test of each option to determine which best meets
campus needs and preferences 

D. Gathering information from schools with different open-campus plans about
the benefits and drawbacks of each plan

Answer and Rationale

NOTE: After clicking on a link, right click and select “Previous View” to go back to original text.

TExES Principal (068) 53


COMPETENCY 008

34. Which TWO of the following are appropriate budget recommendations from a
campus site-based decision-making (SBDM) committee?

A. Use state and federal funds to support programs wherever applicable


B. Offer teachers at the top of the pay scale incentives to retire
C. Create an additional central office administrative position
D. Use data from student assessments to justify program expenditures
E. Change the vendor the district uses to supply food service

Answer and Rationale

COMPETENCY 008

35. The central office informs all principals to expect a 20 percent cut in the
operating budget of each campus. Which of the following steps should each
principal take first to build a budget after a significant budget cut?

A. Committing to cutting all budget lines by the same amount to maintain


equity among programs
B. Providing the campus improvement committee with budget lines that are
essential to the school’s operation
C. Sharing the information with the faculty about the areas to be cut so they
can anticipate reductions
D. Notifying the campus improvement committee of the programs and services
to be cut based on the reduction

Answer and Rationale

COMPETENCY 008

36. A principal is piloting a new student information system on campus. Which of


the following actions by the principal best facilitates teachers’ acceptance of the
system?

A. Providing teachers with time to practice using the system before going live
B. Allowing teachers to learn from each other through collaborative
experimentation
C. Arranging for teachers to be trained by the system’s vendors
D. Highlighting how the system will benefit teachers’ practice

Answer and Rationale

NOTE: After clicking on a link, right click and select “Previous View” to go back to original text.

TExES Principal (068) 54


COMPETENCY 008

37. Which of the following is the most important reason for maintaining a database
of building maintenance and repair records?

A. To prevent emergency repairs from occurring within the facility


B. To promote effective organizational management
C. To encourage more efficient energy consumption by school staff
D. To identify approved vendors when purchases are required

Answer and Rationale

COMPETENCY 009

38. The principal of a high school is reviewing operating costs for the building,
which was built in the 1960s and is not energy efficient. Heating and cooling
costs are especially high. Which of the following is the principal’s best first step
for increasing the building’s energy efficiency?

A. Encouraging the use of fans and space heaters in classrooms 



B. Asking the custodial staff to create a list of areas of greatest energy loss in
the school 

C. Recommending that the school undergo a comprehensive energy audit 

D. Replacing all incandescent lighting in the school with fluorescent lighting

Answer and Rationale

COMPETENCY 009

39. Which of the following is a school principal’s best approach for monitoring
routine custodial tasks?

A. Meeting with the head custodian each week to ask for a summary of the
previous week’s activities 

B. Conducting regular walk-throughs of the building using a checklist to note
the status of all areas of the building 

C. Advising teachers to contact the main office if classrooms and common
rooms are not properly maintained 

D. Making unannounced inspections of different sections of the building and
grounds in the early weeks of each school term

Answer and Rationale

NOTE: After clicking on a link, right click and select “Previous View” to go back to original text.

TExES Principal (068) 55


COMPETENCY 009

40. A local newspaper reporter writes an article about the lack of security at an
elementary school after easily gaining entry through an unlocked side door and
walking through the building without being recognized as an unauthorized
visitor. After reading the article, which of the following actions should the
principal take to make school safety a priority?

A. Reviewing the campus budget to determine whether additional security


cameras can be purchased 

B. Requesting a meeting with the superintendent to ask for funds to hire
security officers 

C. Placing a reminder on each morning announcement that unidentified visitors
should be reported to the office 

D. Meeting with the custodial, office, and teaching staff to review procedures
for securing the building

Answer and Rationale

NOTE: After clicking on a link, right click and select “Previous View” to go back to original text.

TExES Principal (068) 56


Answer Key and Rationales

Question Competency Correct


Number Number Answer Rationales

1 006 B Option B is correct because Texas law prescribes certain legal requirements
and protections for both the terminated employee and the terminating district. It
is most important that Ms. Manning confirm that her actions leading to and at the
point of termination conform to the law. Option A is incorrect because there is
evidence that Ms. Jones has been provided with support that, unfortunately, has
not resulted in improved practice. Option C is incorrect because attending or
not attending new-employee training is not relevant to employee termination.
Option D is incorrect because it seems obvious that Ms. Jones has not adjusted
well based on the documentation provided by the assistant principal.

Back to Question

2 007 D Option D is correct because Ms. Manning has reviewed test scores, grades, and
benchmark assessments (data) and plans to involve teachers in examining the
data and identifying ways to improve student performance (managing groups to
attain campus goals). Option A is incorrect because student diversity is not
identified as a factor influencing students’ weak performance in science.
Option B is incorrect because the teachers are not undergoing instruction, so
the adult learning theory does not apply. Option C is incorrect because Ms.
Manning’s actions are not directed at her personal professional development.

Back to Question

TExES Principal (068) 57


Question Competency Correct
Number Number Answer Rationales

3 001 B Option B is correct because the teachers’ comments indicate that they view
Ms. Manning as an outsider who is unfamiliar with the important issues of the
school and community. As long as they continue to view her in this way, she will
have difficulty establishing herself as a legitimate school leader. Option A is
incorrect because the concern does not relate to the campus vision. Option C is
incorrect because teachers do not indicate that they find Ms. Manning
directionless. Option D is incorrect because the teachers’ comments do not
address barriers.

Back to Question

4 001 C Option C is correct because teachers are a valuable resource for discovering, or
in some cases, uncovering campus needs. Asking for teachers’ solutions also
provides information on their views concerning the direction in which current
efforts are going or should go. This information will prove helpful in fulfilling the
goals of steps 2 and 3. Option A is incorrect because, while staff will learn
about Mr. Ryoko’s expectations for the campus staff, this is not the primary
benefit of his action. Mr. Ryoko is seeking rather than providing information.
Option B is incorrect because Mr. Ryoko is not demonstrating a leadership style
by merely seeking information. Option D is incorrect because reflection is an
introspective process, usually individual and personal, in which teachers examine
their daily practice.

Back to Question

TExES Principal (068) 58


Question Competency Correct
Number Number Answer Rationales

5 007 D Option D is correct because the teams of teachers will use the goals they have
included in the mission statement to identify practices that will achieve those
goals and therefore fulfill the mission. Options A, B and C are incorrect
because these are all possible secondary benefits that may or may not be
realized depending on the principal’s implementation strategies.

Back to Question

6 002 D Option D is correct because a campus mission statement should reflect the
purpose and focus of the entire school community, not just that of the teachers.
Involving other stakeholders will build investment in and commitment to fulfilling
the goals identified in the statement. Options A and B are incorrect because
Mr. Ryoko’s purpose is to involve others in a collaborative process to build shared
commitment. Declaring support for a particular outcome is contrary to the
openness that makes the collaborative process effective, as is delegating
authority to only a few members of the faculty. The contributions of everyone in
the collaborative group, including Mr. Ryoko, should be considered valuable.
Option C is incorrect because the mission and goals of Greenville Elementary
School may not align with those of other district campuses.

Back to Question

TExES Principal (068) 59


Question Competency Correct
Number Number Answer Rationales

7 001 A Option A is correct because the principal should begin by establishing protocols
and commitments developed by the team to guide their work together.
Options B, C and D are incorrect because providing data, celebrating
contributions by stakeholders, and communicating information would happen
at later stages in the process.

Back to Question

8 004 A Option A is correct because the curriculum is being revised to develop and
enhance students’ mathematical skills by the introduction of proven instructional
strategies. The committee will be able to select the appropriate strategies if they
first identify the areas of strength and need. Option B is incorrect because
although expense is an important consideration, it is not relevant if the revision is
not based on student strengths and needs. Option C is incorrect because
curriculum dictates content, resources, and assessment. Option D is incorrect
because instructional materials on hand should not dictate the direction of
curriculum development. A new or revised curriculum often requires new
instructional materials.

Back to Question

TExES Principal (068) 60


Question Competency Correct
Number Number Answer Rationales

9 007 A Option A is correct because it anticipates the concern about lowered test scores
expressed by parents and community members when an earlier curriculum was
revised. Alerting them to the transition period for curriculum changes to show
student growth may allay fears if test scores should drop after the initial
implementation. Options B and C are incorrect because acknowledging
teachers’ efforts and expertise does not address the concerns about mathematics
performance and building long-term support from parents and community
members. Option D is incorrect because requesting input after the curriculum
is completed and submitted to the school board is too late in the process.

Back to Question

10 002 C Option C is correct because demonstrating the link between the students’ areas
of strength and weakness and the strategies for addressing them within the
revised curriculum addresses the parents’ primary concern of maintaining and
building effective mathematics instruction. Option A is incorrect because
parents are likely to be concerned with the effects of the revised curriculum on
the students on their campus, not those on other campuses. Option B is
incorrect because while acknowledgment of the parents’ concerns is a strong
communication strategy, it is not a means for reducing their objections.
Option D is incorrect because, while this strategy does give the parents an
opportunity to be heard, it does not minimize their opposition.

Back to Question

TExES Principal (068) 61


Question Competency Correct
Number Number Answer Rationales

11 002 D Option D is correct because curriculum success is measured by student


performance, and research has consistently shown that when parents are
involved in their children’s education, children perform better, and have more
positive feelings about school and themselves. Options A and B are incorrect
because while they may help a parent increase understanding of the new
curriculum, the most helpful strategy for ensuring success is for parents to
engage with their children in mathematics on a regular basis. Option C is
incorrect because teachers, who observe children within educational contexts
and have access to multiple measures of performance, are able to monitor
student progress more broadly and objectively.

Back to Question

12 006 B Option B is correct because the preparation of general education teachers


focuses mainly on teaching general education students; many general education
teachers benefit from strategies for meeting the needs of students in special
education. Preparing teachers through professional development and increasing
the support of the special education staff are ways that Ms. Winston can meet the
need expressed by the teachers. Option A is incorrect because it appears that
Ms. Winston is open to Mr. Carruthers’s concern and that he feels free to bring it
to her attention. Options C and D are incorrect because neither are concerns
expressed by Mr. Carruthers.

Back to Question

TExES Principal (068) 62


Question Competency Correct
Number Number Answer Rationales

13 003 A Option A is correct because the ARD committee is responsible for determining,
evaluating, and changing a student’s placement. Options B, C and D are
incorrect because by law, none of these are actions or conditions that govern a
change in a student’s placement.

Back to Question

14 003 B Option B is correct because students’ IEPs are the road maps that guide what
and how teachers teach students who receive special education services.
Confirming that teachers are in fact using the IEPs to plan and deliver instruction
is the best way for Ms. Winston to ensure that students’ needs are being met.
Option A is incorrect because teachers may or may not consult the resources in
the library’s professional collection. Option C is incorrect because knowledge of
the laws does not guarantee that the instruction that meets students’ need is
being provided. Option D is incorrect because funding does not affect teachers’
responsibility to comply with the requirements of students’ IEPs.

Back to Question

15 006 D Option D is correct because complying with the accommodation prescribed in a


student’s IEP is not the teachers’ choice: failure to comply puts the teachers and
the district at risk of sanction for noncompliance. Option A is incorrect because
teachers are required to comply with the adaptations prescribed in a student’s
IEP. Option B is incorrect because teacher concerns about the elements in a
student’s IEP are best expressed to the Admission Review Dismissal (ARD)
committee. Option C is incorrect because it is Ms. Winston’s responsibility to
investigate and resolve issues of noncompliance with teachers.

Back to Question

TExES Principal (068) 63


Question Competency Correct
Number Number Answer Rationales

16 007 C Option C is correct because the clearest evidence of inequality of instruction is


the differences in scores between White students and those in the majority of
identified student subgroups, commonly known as the achievement gap.
Option A is incorrect because although the percentage of students receiving
special education or gifted services is a factor to consider, the identification of
students for special education or gifted services does not provide evidence of
inequality of instruction as well as student test scores do. Options B and D
are incorrect because the data are not provided in the TAPR report, so no
conclusions can be drawn with respect to performance by gender or the
assignment of teachers.

Back to Question

17 005 B Option B is correct because the largest and most consistent score difference
between campus and state averages in academic performance is among the
subgroup identified as economically disadvantaged. This supports a conclusion
that socioeconomic status is exerting a greater influence than the other factors.
Option A is incorrect because absentee rates at Grover are comparable to
district and state averages. Option C is incorrect because the average level of
experience of teachers at Grover is comparable to the campus and state levels.
Option D is incorrect because the subgroups identified by ethnicity consistently
score higher in almost all academic areas than students in the subgroup
identified as economically disadvantaged.

Back to Question

TExES Principal (068) 64


Question Competency Correct
Number Number Answer Rationales

18 005 B Option B is correct because the improvement team is seeking professional


development that will have the most impact on improving students' performance.
Large-scale research has found that formative assessment is one of the most
powerful methods for enhancing achievement across the disciplines. Developing
teacher expertise in formative assessment will benefit the performance of
students at all ability levels, in all subgroups, in all subjects, and will have more
impact on student performance than a topic that addresses only the needs of a
specific subgroup. Therefore, it should be the main focus of professional
development for teachers. Option A is incorrect because students are
performing adequately in postsecondary readiness and the use of blended
learning does not necessarily result in improvement in this area. Option C is
incorrect because there is no indication that gifted students are being under-
identified. In fact, the number of students in the gifted program exceeds district
percentages. Option D is incorrect because the report does not provide
information on either the academic performance or social needs of highly mobile
students.

Back to Question

TExES Principal (068) 65


Question Competency Correct
Number Number Answer Rationales

19 007 C Option C is correct because the answer to this question would provide valuable
information to the committee. If retention criteria are applied equally and
consistently, retention rates will be somewhat consistent among student groups
and grade levels; however, there is a significant disparity between the retention
rates of students in different grade levels when compared with district and state
percentages. Option A is incorrect because class sizes in all grades are lower
than both the campus and state averages, and grade levels with low class sizes
have high rates of retention. The data in the report provide the answer to the
question. Option B is incorrect because the data indicate that students in
special education have retention rates that are comparable to if not lower than
district and state percentages. Option D is incorrect because it would be
difficult, if not impossible, to associate specific instructional strategies with
specific teachers, and consequently, with the students of those teachers, given
the multitude of possible strategies and the unlikelihood that a teacher uses one
strategy exclusively. It is also questionable how valuable these data would be for
the committee's purpose.

Back to Question

TExES Principal (068) 66


Question Competency Correct
Number Number Answer Rationales

20 006 A Option A is correct because the ethnicity of the teaching staff at Grover does
not reflect the ethnicity of the student population. Research consistently supports
that students from minority groups perform better in school and remain in school
longer when they have teachers who share their sociological and cultural roots.
Seeking teachers who reflect the minority population and whose presence will
improve the performance of minority students is the action most likely to improve
overall student success. Option B is incorrect because English-language
learners (ELLs) represent a small percentage of the student population and as
helpful as the additional paraprofessionals would be to those students, this action
is unlikely to improve the success of the majority of the students. Option C is
incorrect because the accountability report does not provide data to support the
necessity of a disciplinary officer. Option D is incorrect because the school
currently has a higher percentage of special education teachers on staff than both
the district and the state.

Back to Question

21 001 D Option D is correct because barriers related to gender, ethnicity, and disability
frequently interfere with a campus’s ability to create an inclusive campus culture.
It is the principal’s responsibility to take steps to overcome the barriers, such as
providing targeted professional development. Options A, B and C are incorrect
because promoting staff teamwork, developing a shared vision of the school’s
mission, and ensuring state and federal compliance do not relate to the
professional development topic identified by the principal.

Back to Question

TExES Principal (068) 67


Question Competency Correct
Number Number Answer Rationales

22 001 B Option B is correct because a vision is built on the core values of the campus
and greater communities. Stakeholders cannot begin to craft a vision without first
identifying what those values are. Option A is incorrect because although there
is likely some commonality among visions, each campus vision should be unique
to its campus. Options C and D are incorrect because aligning resources and
developing a plan for the vision would not occur until after the vision is crafted.

Back to Question

23 002 A Option A is correct because people are more likely to support a change if they
have input in the decision-making process. The principal has proactively sought
that input by appointing a committee, conducting a survey, and holding public
meetings. Options B and C are incorrect because the principal’s actions better
demonstrate an understanding of the importance of shared decision making.
Option D is incorrect because the will of the majority may not align with the
campus goals that are driving the changes the principal is proposing.

Back to Question

TExES Principal (068) 68


Question Competency Correct
Number Number Answer Rationales

24 002 B Option B is correct because teams work together to achieve a common goal. If
team members are in conflict, they cannot go forward until attempts are made to
resolve the conflict and gain consensus. The first attempt at gaining consensus is
acknowledging a concern and opening a discussion. Option A is incorrect
because the dissatisfied members have not expressed concern with the entire
curriculum. This solution would also put the committee back to the beginning of
their curriculum selection process. Option C is incorrect because timeliness
does not take priority over the quality of the committee’s recommendation. Time
taken to resolve the conflict and ensure that the committee can unilaterally
support their recommendation is time well spent. Option D is incorrect because
meeting with team members individually promotes division rather than
consensus. Team members’ opinions should be heard and considered by all.

Back to Question

TExES Principal (068) 69


Question Competency Correct
Number Number Answer Rationales

25 003 D Option D is correct because the parents have voiced a strong concern about
this situation and it is most important for the principal to listen to parents
concerns and help them better understand University Interscholastic League
(UIL) guidelines prior to taking any type of action. Option A is incorrect
because the parents are not complaining about the tryout process, so an
additional tryout would likely have the same outcome as the first. Option B is
incorrect because having the parents meet with the coach is a more appropriate
follow-up step after listening and broadening their understanding of the selection
process of competitive programs. Option C is incorrect because Title IX is a law
relating to gender equity in sports programs. Since the parents are not
complaining about gender discrimination, advice from the Title IX coordinator
would not be relevant.

Back to Question

26 004 C Option C is correct because simulations engage students in experiences that


challenge them to make sense of specific concepts and ideas and then apply their
new knowledge in novel situations. The element of active experimentation
inherent in simulations requires students to use the higher-order skills of
analysis, synthesis, and evaluation. Option A is incorrect because simulations
do not require the repetition of routines and procedures. Option B is incorrect
because the application of workplace skills is not a general characteristic of
simulations. Option D is incorrect because although simulations can be
engaging, a simulation that simply engages students without developing their
thinking skills would not serve the teacher’s instructional purpose.

Back to Question

TExES Principal (068) 70


Question Competency Correct
Number Number Answer Rationales

27 005 D Option D is correct because decisions about course offerings are generally
made by a district’s curriculum committee, whose members will explore the
feasibility of all aspects of the request. Option A is incorrect because the
response shows no consideration of a valid request that may enhance course
offerings for all students. Options B and C are incorrect because they bypass
several steps in the procedures for adding a new course of study, procedures that
would likely be directed by the curriculum committee.

Back to Question

28 005 C Option C is correct because working collaboratively to analyze data would occur
first. Option A is incorrect because teachers are unlikely to find admonishment
motivating. Option B is incorrect because financial incentive is inappropriate
because quality instruction is a requirement of a teacher’s job. Such an incentive
would communicate that it is acceptable for teachers to offer anything less than
quality instruction. Option D is incorrect because establishing goals and criteria
for evaluating would be a later step in the process.

Back to Question

TExES Principal (068) 71


Question Competency Correct
Number Number Answer Rationales

29 005 C Option C is correct because an appropriate first step is assessing the extent to
which the school’s existing services meet the needs of the new demographic
groups. After the assessment, the school leadership team can proceed with
strengthening areas of weakness or addressing gaps. Option A is incorrect
because research supports that heterogeneous grouping is more conducive to
improving student achievement than grouping students homogeneously.
Options B and D are incorrect because student success is achieved by
maintaining high standards for all students, not by offering a multiculturally
themed unit or exempting students from challenging courses.

Back to Question

30 005 C Option C is correct because using formative assessment during a study unit to
identify areas for reteaching and retesting is a core element of mastery learning.
Options A, B and D are incorrect because although the actions may enhance
the effectiveness of mastery learning, they are not core elements of the model.

Back to Question

TExES Principal (068) 72


Question Competency Correct
Number Number Answer Rationales

31 006 A, B, D Options A, B and D are correct because surveys on teacher satisfaction


consistently identify a lack of instructional resources, infrequent acknowledgment
of teacher contributions, and exclusion from meaningful decision making as
factors that reduce teacher morale. Any effort the school leadership makes to
address these factors will likely result in improved morale. Option C is incorrect
because teachers cite a lack of involvement in decision making as a reason for
poor morale. Option E is incorrect because providing opportunities for teachers
to collaborate, not compete, is likely to result in improved morale.

Back to Question

32 007 A Option A is correct because the baseline data provided by assessing students
before implementation of the new curriculum allows periodic and frequent
comparison with the scores earned by students during the implementation.
Growth can be demonstrated only by this comparison. Option B is incorrect
because without a basis of comparison, the report card grades will be
meaningless. Additionally, a new curriculum is unlikely to show results after such
a short interval. Option C is incorrect because these data provide only a single
data point and compare last year’s students to this year’s students. Option D is
incorrect because, while qualitative input from stakeholders is helpful, the
school is seeking objective data. Frequent student performance data better meet
their needs.

Back to Question

TExES Principal (068) 73


Question Competency Correct
Number Number Answer Rationales

33 007 D Option D is correct because feedback from schools with open-campus plans will
be a valuable addition to the information the committee has already gathered
and may provide insight into practical aspects that research does not. Option A
is incorrect because the committee has thus far gathered only preliminary
information on the plans. There are several more steps, such as soliciting
feedback and additional data, before endorsing a plan. Option B is incorrect
because although input from students should be considered by the committee, it
should not determine the committee’s recommendation. Option C is incorrect
because pilots are meaningful only when they remain in place for a period of
time. If any of the plans is unsuitable for the campus, a long pilot period may
expose students to risk.

Back to Question

34 008 A, D Options A and D are correct because using state and federal funds to support
programs and justifying expenditures with data from student assessments are
both financially responsible recommendations and retain the focus of the SBDM
committee on campus matters. Options B, C and E are incorrect because they
are all district-level decisions and are not within the SBDM committee's purview.

Back to Question

TExES Principal (068) 74


Question Competency Correct
Number Number Answer Rationales

35 008 B Option B is correct because the campus improvement committee should work
in collaboration with the principal to develop the budget and take the necessary
reduction into consideration. Option A is incorrect because across-the-board
cuts would not take into consideration campus and district goals and focus
initiatives. Option C is incorrect because the principal would notify the staff of
the areas to be cut after the school improvement committee drafts the budget.
Option D is incorrect because deciding which programs and services should be
cut should be made in collaboration with the campus improvement committee.

Back to Question

36 008 D Option D is correct because information about the benefits of the system will
help teachers view the new system as an opportunity for improvement rather
than an unwelcome change. Option A is incorrect because practicing with the
system first may build confidence, but it does not build acceptance. Option B is
incorrect because a student information system is complex, and it would be
difficult and frustrating for teachers to use an experimental method to learn how
to use it. Option C is incorrect because training by the system’s vendor may
help teachers learn how to operate the system, but it does not build acceptance.

Back to Question

TExES Principal (068) 75


Question Competency Correct
Number Number Answer Rationales

37 008 B Option B is correct because a historical record of maintenance data that can be
used to compare facilities or time periods will improve organizational
management and justify costs for preventative maintenance. Option A is
incorrect because emergency repairs will occur within a facility regardless of the
data management system. Option C is incorrect because energy consumption
by school staff is not affected by having a database of district maintenance.
Option D is incorrect because while having an approved list of vendors is
important, the purpose for the database is to justify costs associated with
preventive maintenance.

Back to Question

38 009 C Option C is correct because there are several aspects of energy efficiency, and
a comprehensive audit will identify all problem areas and recommend appropriate
remediation. Option A is incorrect because the heating and cooling system is
only one aspect of energy efficiency, and the solution suggested will likely result
in increased energy costs. Additionally, fans and heaters in classrooms can create
a safety hazard. Option B is incorrect because custodians are not energy
experts and may overlook areas of energy inefficiency that a comprehensive
audit would reveal. Option D is incorrect because upgrading all the lights
addresses only one aspect of energy efficiency and will not affect heating and
cooling costs, which are the primary area of concern.

Back to Question

TExES Principal (068) 76


Question Competency Correct
Number Number Answer Rationales

39 009 B Option B is correct because the principal should personally monitor the
condition of the building, using a tool such as a checklist to ensure thoroughness
and to maintain a record of the observations. Option A is incorrect because this
limits the principal’s awareness to only those issues the head custodian shares
during the meeting. Option C is incorrect because this puts an additional
responsibility on teachers and will depend on teachers’ individual assessments of
acceptable and unacceptable building conditions. Option D is incorrect because
the principal’s monitoring should be frequent and ongoing, not limited to the
early weeks of each term.

Back to Question

40 009 D Option D is correct because custodial, office and teaching staff are all
responsible for executing the school's security plan, e.g., making sure doors are
locked from the outside, following proper procedures for admitting visitors to the
building, and alerting the office to unauthorized visitors. Options A and B are
incorrect because these expenses are not justified until the measures in place
have been deemed ineffective. At this point, the measures cannot be deemed
ineffective because they have not been properly followed. Option C is incorrect
because repeated reminders without action lose their meaning and the reminders
may lead students to feel that they are constantly unsafe.

Back to Question

TExES Principal (068) 77


Study Plan Sheet

STUDY PLAN
What What
material do material do
I have for I need for Where can Dates
Content How well do studying studying I find the planned for
covered I know the this this materials study of Date
on test content? content? content? I need? content Completed

TExES Principal (068) 78


Preparation Resources
The resources listed below may help you prepare for the TExES test in this field.
These preparation resources have been identified by content experts in the field to
provide up-to-date information that relates to the field in general. You may wish to
use current issues or editions to obtain information on specific topics for study
and review.

ORGANIZATIONS
American Educational Research Association
1430 K Street, NW
Suite 1200
Washington, DC 20005

Association for Supervision and Curriculum Development


1703 North Beauregard Street
Alexandria, VA 22311-1714
800-933-2723

Learning Forward
504 South Locust Street
Oxford, OH 45056
800-727-7288

National Association of Elementary School Principals (NAESP)


1615 Duke Street
Alexandria VA 22314
1-800-386-2377

National Association of Secondary School Principals (NASSP)


1904 Association Drive
Reston VA 20191-1537
703-860-0200

Phi Delta Kappa International


320 W. Eighth Street, Suite 216
Bloomington, IN 47404
812-339-1156

Texas Association for Supervision and Curriculum Development


1601 Rio Grande, Suite 451
Austin, TX 78701
512-477-8200

Texas Association of School Administrators


406 E. 11th Street
Austin, TX 78701
512-477-6361

TExES Principal (068) 79


Texas School Administrators’ Legal Digest
1601 Rio Grande, Suite 455
Austin, TX 78701
512-478-2113

JOURNALS AND PERIODICALS


ASCD Education Update, Association for Supervision and Curriculum Development.
Communicator, National Association of Elementary School Principals.
Educational Administration Quarterly, University Council for Educational
Administration.
Educational Leadership, Association for Supervision and Curriculum Development.
Educational Researcher, American Educational Research Association.
INSIGHT, Texas Association of School Administrators.
Instructional Leader, Texas Elementary Principals and Supervisors Association.
Newsleader, National Association of Secondary School Principals.
Phi Delta Kappan, Phi Delta Kappa.
Principal, National Association of Elementary School Principals.
Principal Leadership, National Association of Secondary School Principals.

OTHER RESOURCES
Aguilar, E. (2013). The art of coaching: Effective strategies for school
transformation. San Francisco, Calif.: Jossey-Bass.
Bambrick-Santoyo, P. (2012). Leverage leadership: A practical guide to building
exceptional schools. San Francisco, Calif.: Jossey-Bass.
Daresh, J. C. (2006). Beginning the principalship: A practical guide for new school
leaders, Third Edition. Thousand Oaks, Calif.: Corwin Press.
Davis, S., Darling-Hammond, L., LaPointe, M., & Meyerson, D. (2005). School
leadership study: Developing successful principals. Stanford, Calif.: Stanford
University, Stanford Educational Leadership Institute.
Deal, T. E., & Peterson, K. D. (2009). Shaping school culture: Pitfalls, paradoxes
and promises, Second Edition. San Francisco, Calif.: Jossey-Bass.
DuFour, R. & Marzano, R. J. (2011). Leaders of learning: How district, school, and
classroom leaders improve student achievement. Bloomington, Ind.: Solution Tree
Press.
Fullan, M. (2014). The principal: Three keys to maximizing impact. San Francisco,
Calif.: Jossey-Bass.
Glickman, C. D., Gordon, S. P., & Ross-Gordon, J. M. (2013). SuperVision and
instructional leadership: A developmental approach, Ninth Edition. New York, N.Y.:
Pearson Education.

TExES Principal (068) 80


Grogan, M. (Ed.) (2013). The Jossey-Bass reader on educational leadership, Third
Edition. San Francisco, Calif.: Jossey-Bass.
Hodkinson, A. (2015). Key issues in special educational needs and inclusion,
Second Edition. Thousand Oaks, Calif.: Sage.
Kowalski, T. J. (2011). Case studies on educational administration, Sixth Edition.
Upper Saddle River, N.J.: Pearson.
Martin, L. E., Kragler, S., Quatroche, D. J., & Bauserman, K. L. (Eds.). (2015).
Handbook of professional development in education: Successful models and
practices, PreK–12. New York, N.Y.: The Guilford Press.
Marzano, R. (2003). What works in schools: Translating research into action.
Alexandria, Va.: Association for Supervision and Curriculum Development.
Marzano, R., Waters, T., & McNulty, B. (2005). School leadership that works.
Alexandria, Va.: Association for Supervision and Curriculum Development.
McDonald, J. P. & The Cities and Schools Research Group. (2014). American school
reform: What works, what fails, and why. Chicago, Ill.: University of Chicago.
McTighe, J. & Wiggins, G. (2013). Essential questions: Opening doors to student
understanding. Alexandria, Va.: Association for Supervision and Curriculum
Development.
Osborne, A.G. & Russo, C.J. (2014). Special education and the law: A guide for
practitioners, Third Edition. Thousand Oaks, Calif.: Corwin.
Sadeghi, L. & Callahan, K. (Eds.). (2015). Educational leadership in action: A
casebook for aspiring administrators. New York, N.Y.: Routledge.
Schlechty, P. C. (2011). Engaging students: The next level of working on the work.
San Francisco, Calif.: Jossey-Bass.
Schlechty, P. C. (2009). Leading for learning: How to transform schools into
learning organizations. San Francisco, Calif.: Jossey-Bass.
Schmoker, M. (2006). Results now: How we can achieve unprecedented
improvements in teaching and learning. Alexandria, Va.: Association for Supervision
and Curriculum Development.
Schmoker, M. (2011). Focus: Elevating the essentials to radically improve student
learning. Alexandria, Va.: Association for Supervision and Curriculum Development.
Sergiovanni, T. J. (2007). Rethinking leadership: A collection of articles, Second
Edition. Thousand Oaks, Calif.: Corwin Press.
Sergiovanni, T. J. (2005). Strengthening the heartbeat: Leading and learning
together in schools. San Francisco, Calif.: Jossey-Bass.
Tschannen-Moran, M. (2014). Trust matters: Leadership for successful schools,
Second Edition. San Francisco, Calif.: Jossey-Bass.
Ubben, G. C., Hughes, L. W., Norris, C. J. (2015). The principal: Creative leadership
for excellence in schools, Eighth Edition. Upper Saddle River, N.J.: Pearson.
Walsh, J., Kemerer, F., & Maniotis, L. (2014). The educator’s guide to Texas school
law, Eighth Edition. Austin, Texas: University of Texas Press.

TExES Principal (068) 81


Whitaker, T. (2002). What great principals do differently: Eighteen things that
matter most, Second Edition. Larchmont, N.Y.: Eye on Education.
Wiggins, G. & McTighe, J. (2013). The understanding by design guide to creating
high quality units. New York, N.Y.: Routledge.
Zepeda, S. J. (2013). The principal as instructional leader: A practical handbook,
Third Edition. New York, N.Y.: Routledge.

ONLINE RESOURCES
Education Commission of the States — www.ecs.org
Education Law — Guide to Education Law — www.hg.org/edu.html
National Center on Educational Outcomes — www.cehd.umn.edu/nceo
Texas Administrative Code — www.sos.state.tx.us/tac/index.shtml
Texas Constitution and Statues/Texas Education Code —
www.statutes.legis.state.tx.us
Texas Education Agency — www.tea.texas.gov
Texas Project FIRST — www.texasprojectfirst.org
U.S. Department of Education — www.ed.gov

TExES Principal (068) 82

You might also like